Juste de la logique...

Soit f une application de IN* dans IN* telle que pour tout entier naturel n non nul, on a : f(f(n))<f(n+1).
Montrer que pour tout entier naturel n, on a : f(n)=n.

Réponses

  • Bonjour,

    Regardez le sixième problème des IMO 1977 à Belgrade.
  • à SXB pourquoi tu as titré "juste de la logique"? Il me semble que ce genre de devinettes se résout avec pas spécialement plus "de logique" qu'un problème de maths "habituel" non?

    Sinon, je veux pas casser l'énigme donc je ne mets pas les "si; donc; ..." mais propose un enchainement comme indice:

    $t+1\geq a+1$ et $a+1>f(t+1)$ =>
    $ a+1 >f(t+1)>f(f(t))$ =>
    $ a>f(f(t))$ ( :D=> :D ) $a>f(t) $ ...

    Ah, remarque, peut-être que le $\exists$ qui intervient d'une manière précise, tu considères ça comme de la logique?

    Maintenant que j'ai écrit ça, ça devient de la logique de rédiger :D
    Aide les autres comme toi-même car ils sont toi, ils sont vraiment toi
  • @ SXB :
    f(n)=n+a est aussi solution... Est ce que f doit etre bijective?
  • Avant d'aller au dodo, je retire ce que j'ai dit et rends hommage à SXB qui a raison (pédagogiquement, et en les temps modernes): c'est bien un exo qu'on peut considérer comme emprunt d'une dose de logique (je viens de lire le deuxième post) pour des ados, puisqu'ils doivent, pour le résoudre, prendre le plus entier $n$ tel qu'il existe un entier p avec $p\geq n$ et $n>f(p)$ (le fait ensuite que si $f\neq id$ alors il existe un tel entier n est aisé je crois pour ces ados)

    Enfin du moins c'est l'indice que j'ai donné, mais il n'est pas clair qu'il existe une solution calculatoire d'un autre style.

    Toutes mes excuses SXB...
    Aide les autres comme toi-même car ils sont toi, ils sont vraiment toi
  • nico69 écrivait:
    > f(n)=n+a est aussi solution...

    Pas vraiment : si \(f(n) = n+a\), alors \(f(f(n)) = n+2a\) et \(f(n+1) = n+1+a\) ; la condition \(f(f(n))<f(n+1)\) impose donc \(a<1\), c'est-à-dire \(a=0\).
  • Non, nico69, $f(n)=n+a$ n'est pas solution, sauf si $a<1$, i.e. $a=0$ (car $a$ est un entier positif, je suppose?).

    (On a $f(f(n))=f(n+a)=n+2a$ et $f(n+1)=n+1+a$, et donc on doit avoir $2a<a+1$)
  • à nico: (bon lol gb t'a répondu).

    Bon je mets juste ma solution de ce matin rédigée, mais après je la colorerai en blanc, c'est juste pour vérifier que je ne fais perdre de temps à personne avec l'indice:

    Soit $n$ le premier entier tel que $f(n)\neq n$. Admettons qu'il s'écrive $p+1$. ***

    alors $p=f(p)$ et $f(p+1)>f(f(p))=p$ donc $f(n)>n$. Soit $a:=f(n)$. alors $f(n+1)>f(a)$ et pourtant $a\geq n+1$

    Il s'ensuit qu'il existe $a$ et $b$ tel que $a\geq b$ et $b>f(a)$.

    Soit alors $n$ le premier entier tel qu'il existe $t$ avec $t\geq n$ et $n>f(t)$. En écrivant $n=m+1$ et $t=u+1$, on obtient que $u\geq m$. Par ailleurs, $m+1>f(u+1)>f(f(u))$ donc $m>f(f(u))$ et comme $m+1$ est le plus petit blabla, c'est que $m>f(u)$, mais alors contradiction (car $u\geq m$)


    Dans ***, j'ai triché en supposant que n n'est pas 1, mais bon ça laisse un trou à boucher :D et de toute façon ça me fait tellement mal au crâne...
    Aide les autres comme toi-même car ils sont toi, ils sont vraiment toi
  • Ne lire que si vous n'avez pas envie de chercher (et en plus c'est sans garantie, c'est juste que je voulais vérifier que l'indice posté à la va vite ne déconnait pas), d'où le jaune (il suffit de sélectionner pour faite apparaitre...
    Aide les autres comme toi-même car ils sont toi, ils sont vraiment toi
  • si on pose g(n) = n+1 alors on a pour tout n de |N*

    (f o f)(n) <(f o g)(n)

    il est clair que f(n) est différent de g(n) (inégalité strict), alors

    Soit f(n) > g(n), mais comme (f o f)(n) <(f o g)(n) cela signifie que f(n) est strictement décroissante, ce qui est impossible sur N

    Soit f(n) < g(n) => f(n)<n+1 => f(n) = n ou f(n)=min(IN) or f(n) est différent de 0. Aussi f(n) = n.

    j'ai bon ?
  • ça manque un peu de rigueur ce que j'ai mis

    1 - f(n) ne saurait être constante ou partiellement constante (à partir de n0 f(n) est constante) sinon l'inégalité strict ne peut pas s'appliqué en tout n

    2 - une faute de frappe, il faut lire

    Soit f(n) < g(n) => f(n) < n+1 => f(n) = n.
  • Bonjour,

    Souvenirs, souvenirs: par exemple... http://www.les-mathematiques.net/phorum/read.php?5,328155,328155#msg-328155
    Cet IMO est proposé au moins une fois chaque année sur notre forum.

    Amicalement.
  • Bonjour à tous,

    > mamane

    J'ai du mal à te suivre car tu commences par "pour tout n", mais ensuite tu ne distingues que deux cas f<g ou f>g. Le sens de l'inégalité peut très bien varier selon les valeurs de n. Mais bon, peut-être que quelque chose m'échappe...

    Je propose ceci :

    L'ensemble des images possède un plus petit élément f (k).

    Supposons que k soit différent de 1.

    Alors f(f(k-1)) < f(k) ce qui contredit le fait que f(k) soit la plus petite image.

    Donc f(1) est la plus petite image.

    Le même raisonnement montre en outre que l'image f(1) n'est atteinte qu'une fois :

    s'il existait k différent de 1 tel que f(k) = f(1), alors f(f(k-1)) < f(k) c'est à dire f(1) ce qui contredit le fait que f(1) soit la plus petite image.

    Ecartons f(1) de l'ensemble des images et soit f(m) la plus petite image restante.

    Nous savons que m est différent de 1 puisque f(1) a été écarté.

    Supposons que m soit différent de 2.

    Alors f(f(m-1)) < f(m), ce qui contredit le fait que f(m) soit la plus petite image restante puisque nous savons que f(m-1) ne peut pas être égal à f(1).

    Le même raisonnement qu'au-dessus nous montre que l'image f(2) n'est atteinte qu'une fois.

    On peut poursuivre avec f(3), f(4) etc.

    On a donc un isomorphisme d'ordre de N sur f(N) : si a < b, alors f(a) < f(b)

    On observe que, pour tout n, f(n) ne peut pas être plus petit que n.

    Supposons qu' il existe p avec f(p) différent de p.

    Donc f(p) > p

    Donc f(f(p) < f(p+1) (d'après l'énoncé), mais aussi f(f(p) > f(p) (comme nous l'avons montré).

    Or, dans la suite croissante des images, il n'y a aucune place entre f(p) et f(p+1).

    Donc, quel que soit p, f(p) = p.

    Aldo.
  • En encre foncée,voici un découpage:


    1) prouver que $\forall x,y:x\geq y\to f(x)\geq y$ (aide montrer que l'ensemble des $x$ tels que $\exists y: y\geq x$ et $x>f(y)$ n'a pas de minimum

    2) en déduire le résultat

    Ce découpage rend l'exo facile, l'aspect "logique" de la question étant la considération de l'ensemble $A:=\{x/\exists y: blabla \}$
  • Dernier point d'interrogation : pourquoi a-t-on écarté le zéro ?

    Aldo.
Connectez-vous ou Inscrivez-vous pour répondre.